55
$\begingroup$

I came on the following multiple integral while renormalizing elliptic multiple zeta values: $$\int_0^1\cdots \int_0^1\int_1^\infty {{1}\over{t_n(t_{n-1}+t_n)\cdots (t_1+\cdots+t_n)}} dt_n\cdots dt_1.$$ Only the variable $t_n$ goes from $1$ to $\infty$, the others all go from $0$ to $1$. Numerically, I seem to be getting a rational multiple of $\pi^n$. I would like to prove this. Has anyone ever seen an integral like this before?

Edit: After reworking out why I thought it would be a power of $\pi$, I now have a different integral, which numerically really does seem to give a rational multiple of $\zeta(n)$ for each $n$ (though I have only been able to go up to n=4 numerically). I want to integrate $1/(z_1\cdots z_n)$ over the part of the simplex $0\le z_n\le \cdots \le z_1\le 1$ in which $|z_i -z_{i+1}|>\varepsilon$, then let $\varepsilon\rightarrow 0$. The integral is $$\int_{n\epsilon}^{1-\varepsilon}\int_{(n-1)\varepsilon}^{z_1-\epsilon}\cdots \int_{\varepsilon}^{z_{n-1}-\varepsilon} {{1}\over{z_1\cdots z_n}} dz_n\cdots dz_1.$$ It actually diverges when $\varepsilon\rightarrow 0$, but it can be regularised like ordinary multizeta values by computing the integral as a power series in $ln(\varepsilon)$ and $\varepsilon$ and then taking its constant term. It's related to the previous integral by the variable change $z_1=\varepsilon(t_1+\cdots+t_n),\ldots,z_n=\varepsilon t_n$, but the new bounds of the integral form an $(n+1)$-angled polyhedron, not a cube.

$\endgroup$
26
  • 16
    $\begingroup$ For $n=3$ the integral seems to be $5\zeta(3)/24$. $\endgroup$ Apr 17, 2017 at 20:02
  • 2
    $\begingroup$ For $n=4$ the same method produces (after about 3 minutes) $0.081724116967726819510127555243991754831$, which doesn't seem to be a simple multiple of $\pi^4$ --- but I don't know how many of those digits are trustworthy coming from an improper triple integral . . . $\endgroup$ Apr 18, 2017 at 4:52
  • 5
    $\begingroup$ Some say the integral will end in pi, some say it's not so nice. From what I've tasted of periods, I hold with those who favor pi. But only if n is twice An integer or else I think, The integral won't be so nice, but still pretty great, and zetas sould suffice. $\endgroup$
    – Marty
    Apr 18, 2017 at 6:24
  • 3
    $\begingroup$ @FedorPetrov No reason to ask Noam for a computation we can easily do -- the first few are 1/12, 4/49, 17/208, 55/673, 237/2900, 1240/15173, 2717/33246 . None of these are particularly accurate given their denominator. $\endgroup$ Apr 18, 2017 at 16:29
  • 3
    $\begingroup$ @SylvainJULIEN I'm pretty sure that this is not the case. I too computed an approximation (using quad from mpmath), which matches Noam Elkies' calculation on 18 decimal digits, while your number matches only 7 digits. $\endgroup$ Apr 18, 2017 at 23:12

4 Answers 4

25
$\begingroup$

Let's record what is possible, so far. The case $n=2$, from the comments: $$\int_0^1\frac{dy}{x(x+y)}=\frac1{x^2}\int_0^1\frac{dy}{1+\frac{y}x}=\int_0^1\sum_{k\geq0}\frac{(-1)^ky^k}{x^{k+2}}dy=\sum_{k\geq0}\frac{(-1)^k}{k+1}\frac1{x^{k+2}}.$$ Hence, $$\int_1^{\infty}\sum_{k\geq0}\frac{(-1)^k}{k+1}\frac{dx}{x^{k+2}}=\sum_{k\geq0}\frac{(-1)^k}{(k+1)^2}=\frac{\zeta(2)}2.$$

The case $n=3$: $$\begin{aligned}\int_0^1\frac{dz}{x(x+y)(x+y+z)}&=\frac1{x(x+y)^2}\int_0^1\frac{dz}{1+\frac{z}{x+y}}\\&=\frac1x\sum_{k\geq0}\int_0^1\frac{(-1)^kz^kdz}{(x+y)^{k+2}}\\&=\frac1x\sum_k\frac{(-1)^k}{(k+1)(x+y)^{k+2}}.\end{aligned}$$ Hence, $$\frac1x\int_0^1\sum_k\frac{(-1)^kdy}{(k+1)(x+y)^{k+2}}=\frac1x\sum_k\frac{(-1)^k}{(k+1)^2}\left[\frac1{x^{k+1}}-\frac1{(x+1)^{k+1}}\right].$$ Now, integrate with respect to $x$ (standard): $$\int_1^{\infty}\frac{dx}{x^{k+2}}=\frac1{k+1} \qquad \text{and} \qquad \int_1^{\infty}\frac{dx}{x(x+1)^{k+1}}=\sum_{j=k+1}^{\infty}\frac1{j\cdot 2^j}.$$ Therefore, we compute the two series: $$\sum_{k=1}^{\infty}\frac{(-1)^{k-1}}{k^3}=\frac34\zeta(3) \qquad \text{and} \qquad \sum_{k=1}^{\infty}\frac{(-1)^{k-1}}{k^2}\sum_{j=k}^{\infty}\frac1{j\cdot 2^j}=\frac{13}{24}\zeta(3).$$ We arrived at the valued predicted by Peter Mueller, $$\int_1^{\infty}\int_0^1\int_0^1\frac{dz\,dy\,dx}{x(x+y)(x+y+z)}=\frac5{24}\zeta(3).$$

Caveat. One may anticipate higher values of $n$ to scale up the challenge.

UPDATE. Regarding Fedor's question, one contention is a follows: the sum in question is a weight 3 polylog, so it is a rational combination of $\zeta(3), \zeta(2)\log 2$ and $\log^3(2)$. Since a large numerical agreement verifies equality with only $\frac{13}{24}\zeta(3)$, it must be the exact evaluation.

UPDATE. I like to address the request from GH from MO directed to Agno. Then, a direct answer to Fedor's question.

This time, we start integrating with respect to $x$: $$\int_1^{\infty}\frac{dx}{x(x+y)(x+y+z)} =\frac{y\,\log(1+y)+z\,\log(1+y)-y\,\log(1+y+z)}{yz(y+z)}.$$ Next, integrate in the variable $y$: $$\int_0^1\frac{y\,\log(1+y)+z\,\log(1+y)-y\,\log(1+y+z)}{yz(y+z)}\,dy =\frac{\text{Li}_2(z+2)-\text{Li}_2(z+1)-\text{Li}_2(2)}z;$$ where $\text{Li}_2(z)$ is the dilogarithm function $$\text{Li}_2(z)=\int_1^z\frac{\log t}{1-t}\,dt.$$ Finally, we integrate in the last variable $z$: $$\begin{align} \int_0^1\frac{\text{Li}_2(z+2)-\text{Li}_2(z+1)-\text{Li}_2(2)}z\,dz &=\int_0^1\left(\int_1^{z+2}\frac{\log t}{1-t}\,dt-\int_1^{z+1}\frac{\log t}{1-t}\,dt-\int_1^2\frac{\log t}{1-t}\,dt \right)\frac{dz}z \\ &=\int_0^1\left(\int_{z+1}^{z+2}\frac{\log t}{1-t}\,dt-\int_1^2\frac{\log t}{1-t}\,dt \right)\frac{dz}z \\ &=\int_0^1\left(\int_2^{z+2}\frac{\log t}{1-t}\,dt-\int_1^{z+1}\frac{\log t}{1-t}\,dt\right)\frac{dz}z \\ &=\int_2^3\frac{\log t}{1-t}\left(\int_{t-2}^1\frac{dz}z\right)dt-\int_1^2\frac{\log t}{1-t}\left(\int_{t-1}^1\frac{dz}z\right)dt \\ &=\int_2^3\frac{\log t\,\log(t-2)}{t-1}\,dt-\int_1^2\frac{\log t\,\log(t-1)}{t-1}\,dt \\ &=\int_0^1\frac{\log (t+2)\,\log t}{t+1}\,dt-\int_0^1\frac{\log(t+1)\,\log t}t\,dt \\ &=\int_0^1\frac{\log (t+2)\,\log t}{t+1}\,dt+\frac34\zeta(3). \end{align}$$ For the first integral in the last equality, write $\log(t+2)=\log 2+\log(1+\frac{t}2)$ and apply Taylor series: $$\begin{align} \int_0^1\frac{\log (t+2)\,\log t}{t+1}\,dt &=\log 2\int_0^1\frac{\log t}{t+1}\,dt+\sum_{n\geq1}\frac{(-1)^{n-1}}{2^nn}\int_0^1\frac{t^n\log t}{t+1}\,dt \\ &=-\frac12\zeta(2)\,\log2+\sum_{n\geq1}\frac{(-1)^{n-1}}{2^nn}\int_0^1\frac{t^n\log t}{t+1}\,dt \\ &=-\frac12\zeta(2)\,\log2+\sum_{n\geq1}\frac{(-1)^{n-1}}{2^nn}\left[\frac{(-1)^{n-1}}2\zeta(2)+(-1)^{n-1}\sum_{k=1}^n\frac{(-1)^k}{k^2}\right] \\ &=-\frac12\zeta(2)\,\log2+\frac12\zeta(2)\sum_{n\geq1}\frac1{2^nn}+\sum_{n\geq1}\frac1{2^nn}\sum_{k=1}^n\frac{(-1)^k}{k^2} \\ &=-\frac12\,\zeta(2)\,\log2+\frac12\,\zeta(2)\,\log2+\sum_{n\geq1}\frac1{2^nn}\sum_{k=1}^n\frac{(-1)^k}{k^2} \\ &=\sum_{k\geq1}\frac{(-1)^k}{k^2}\sum_{n=k}^{\infty}\frac1{2^nn}. \end{align}$$ The above derivations indicate we do not need Agno's $\log(e^x\pm1)$ integrals, instead we got Robert Z's $\log$-integral which sends us to his useful link evaluating as $\frac{13}{24}\zeta(3)$.

$\endgroup$
11
  • 4
    $\begingroup$ Why $\sum_{k=1}^{\infty}\frac{(-1)^{k-1}}{k^2}\sum_{j=k}^{\infty}\frac1{j\cdot 2^j}=\frac{13}{24}\zeta(3)?$ $\endgroup$ Apr 18, 2017 at 11:18
  • 1
    $\begingroup$ After inserting Noam's high precision calculation from above, the inverse symbolic calculator answered "Wow, really found nothing" :( $\endgroup$ Apr 18, 2017 at 13:15
  • 3
    $\begingroup$ I would also like to see the crucial step pointed out by Fedor Petrov. $\endgroup$
    – GH from MO
    Apr 18, 2017 at 16:46
  • 1
    $\begingroup$ For what it's worth. After some integration by parts and substitution of variables, I found that $\sum_{k=1}^{\infty}\frac{(-1)^{k-1}}{k^2}\sum_{j=k}^{\infty}\frac1{j\cdot 2^j}$ could also be expressed as: $$\int_{ln(2)}^{\infty} (\ln(e^x+1)-x)\,\ln(e^x-1)\, dx$$ or even simpler, it would imply that: $$\frac{13}{24}\zeta(3)=-\int_{0}^{\ln(2)} \ln(e^x+1)\,\ln(e^x-1) \,dx$$ Note that numerically the following related integral also seems true: $$\int_{0}^{\infty} \left(\ln(e^x+1)-x\right)\ln(e^x-1) dx = \frac18\,\zeta(3)$$ $\endgroup$
    – Agno
    Apr 25, 2017 at 12:34
  • 7
    $\begingroup$ From Agno's comment, by letting $t=e^x-1$, we obtain $$-\int_{0}^{\ln(2)} \ln(e^x+1)\,\ln(e^x-1) \,dx=\int_0^1 \frac{\log \left(\frac{1}{t}\right) \log (t+2)}{t+1} \, dt$$ which is equal to $\frac{13}{24}\zeta(3)$ by math.stackexchange.com/questions/1344455/… $\endgroup$
    – Robert Z
    Apr 25, 2017 at 17:38
13
$\begingroup$

You can easily reduce to a double integral and in a few seconds you obtain 0.0817241169677268267134627567961242656233303062217211895... (all digits correct, I checked at higher accuracy), but still cannot recognize it.

$\endgroup$
1
  • 5
    $\begingroup$ I take it this is for $n=4$. $\endgroup$ Apr 24, 2017 at 0:36
13
$\begingroup$

The following is a somewhat simplified version of the proof by T. Amdeberhan, Agno, and Robert Z that $$\int_0^1 \int_0^1 \int_1^\infty \frac{dx\,dy\,dz}{x(x+y)(x+y+z)}=\frac{5}{24}\zeta(3).$$ We integrate with respect to $x$ first, then with respect to $y$: \begin{align*}\int_0^1\int_1^\infty \frac{dx\,dy}{x(x+y)(x+y+z)}&=\int_0^1\left(\frac{\log(1+y)}{yz}-\frac{\log(1+y+z)}{(y+z)z}\right)dy\\ &=\int_0^1\frac{\log(1+t)}{tz}\,dt-\int_z^{1+z}\frac{\log(1+t)}{tz}\,dt\\ &=\int_0^z\frac{\log(1+t)}{tz}\,dt-\int_1^{1+z}\frac{\log(1+t)}{tz}\,dt.\\ \end{align*} The integral of the right hand side with respect to $z$ equals, by Fubini, \begin{align*}&=\int_0^1\int_0^z\frac{\log(1+t)}{tz}\,dt\,dz-\int_0^1\int_1^{1+z}\frac{\log(1+t)}{tz}\,dt\,dz\\ &=\int_0^1\int_t^1\frac{\log(1+t)}{tz}\,dz\,dt-\int_1^2\int_{t-1}^1\frac{\log(1+t)}{tz}\,dz\,dt\\ &=-\int_0^1\frac{\log(t)\log(t+1)}{t}\,dt+\int_1^2\frac{\log(t-1)\log(t+1)}{t}\,dt\\ &=-\int_0^1\frac{\log(t)\log(t+1)}{t}\,dt+\int_0^1\frac{\log(t)\log(t+2)}{t+1}\,dt. \end{align*} On the right hand side, the second integral equals $-\frac{13}{24}\zeta(3)$ by this MSE post, while the first integral equals \begin{align*}\int_0^1\frac{\log(t)\log(t+1)}{t}\,dt &=\int_0^1\frac{\log(t)}{t}\sum_{n=1}^\infty\frac{(-1)^{n-1}t^n}{n}\,dt\\ &=\sum_{n=1}^\infty\frac{(-1)^{n-1}}{n}\int_0^1 t^{n-1}\log(t)\,dt=\\ &=\sum_{n=1}^\infty\frac{(-1)^n}{n^3}=-\frac{3}{4}\zeta(3). \end{align*} To summarize, $$\int_0^1 \int_0^1 \int_1^\infty \frac{dx\,dy\,dz}{x(x+y)(x+y+z)}=\frac{3}{4}\zeta(3)-\frac{13}{24}\zeta(3)=\frac{5}{24}\zeta(3).$$

$\endgroup$
5
$\begingroup$

As suggested by მამუკა ჯიბლაძე, for n=2,3 there is indeed an interesting connection of $$I_n=\int\limits_0^1\cdots \int\limits_0^1 dt_1\cdots dt_{n-1}\int\limits_1^\infty dt_n \frac{1}{t_n(t_n+t_{n-1})\cdots(t_n+\cdots +t_1)}$$ with the Beukers-type integrals. Using the Feynman parametrization $$\frac{1}{A_1A_2\cdots A_n}=(n-1)!\int\limits_0^1\cdots \int\limits_0^1\frac{\delta\left(1-\sum\limits_{i=1}^n x_i\right)dx_1\cdots dx_n}{(x_1A_1+x_2A_2+\cdots +x_nA_n)^n},$$ we get after the parametrization and subsequent trivial integration over $t_n$: $$I_n=\frac{(n-1)!}{n-1}\int\limits_0^1\cdots \int\limits_0^1 \frac{dx_1\cdots dx_n dt_1\cdots dt_{n-1}\;\delta(1-x_1-\cdots-x_n)}{[1+t_{n-1}(x_{n-1}+\cdots x_1)+\cdots +t_1x_1]^{n-1}}.$$ In particular, for n=2 we get immediately $$I_2=\int\limits_0^1\int\limits_0^1 \frac{dx dt}{1+xt}=\frac{1}{2}\zeta(2),$$ and for n=3 we get $$I_3=\int\limits_0^1\cdots\int\limits_0^1 \frac{dx_1 dx_2 dx_3dt_1 dt_2\;\delta(1-x_1-x_2-x_3)}{[1+t_2(x_1+x_2)+t_1x_1]^2}=$$ $$\iint\limits_0^1\frac{dx_1dx_2dx_3\delta(1-\sum\limits_{i=1}^3 x_i)}{x_1(x_1+x_2)}\left [\ln{(1+x_1+x_2)}+\ln{(1+x_1)}-\ln{(1+2x_1+x_2)}\right]=$$ $$\iiint\limits_0^1\frac{dx_1dx_2dx_3\;\delta(1-\sum\limits_{i=1}^3 x_i)}{x_1(1-x_3)}\left [\ln{(2-x_3)}+\ln{(1+x_1)}-\ln{(2+x_1-x_3)}\right].$$ Making $x=x_1,y=1-x_3,z=x_2$ change of variables in the integral, and performing integration over $z$ thanks to the $\delta$-function, we get $$I_3=\iint\limits_0^1\frac{dx dy}{xy}\left[\ln{(1+x)}+\ln{(1+y)}-\ln{(1+x+y)}\right]\theta(y-x)=$$ $$ \frac{1}{2}\iint\limits_0^1\frac{dx dy}{xy}\left[\ln{(1+x)}+\ln{(1+y)}-\ln{(1+x+y)}\right], \tag{1}$$ where in the last step we used the symmetry of the integrand and $\theta(y-x)+\theta(x-y)=1$. This result can be represented in the form $$I_3=\frac{1}{2}\iiint\limits_0^1\frac{dx dy dz}{1+x+y+xyz}. \tag{2}$$ Note the resemblance with Beukers integrals $$ \iiint\limits_0^1\frac{dx dy dz}{1-z+xyz}=2\zeta(3),\;\; \iiint\limits_0^1\frac{dx dy dz}{1+xyz}=\frac{3}{4}\iiint\limits_0^1\frac{dx dy dz}{1-xyz}=\frac{3}{4}\zeta(3).$$ Analogously, for $I_4$ we get $$I_4=\iiint\limits_0^1\frac{dx dy dz}{xyz}\left[\ln{(1+x)}+\ln{(1+y)}+ \ln{(1+z)}+\ln{(1+x+y+z)}-\right . $$ $$ \left . \ln{(1+x+y)}- \ln{(1+x+z)}-\ln{(1+y+z)}\right]\theta(z-x)\theta(y-z).$$ $\theta(z-x)\theta(y-z)$ term is a consequence of integration with the help of the $\delta$-function and reflects $x_2=z-x>0$ and $x_3=y-z>0$ conditions. After symmetrization, we finally get $$I_4=\frac{1}{6}\iiint\limits_0^1\frac{dx dy dz}{xyz}\left[\ln{(1+x)}+\ln{(1+y)}+ \ln{(1+z)}+\ln{(1+x+y+z)}-\right . $$ $$ \left . \ln{(1+x+y)}- \ln{(1+x+z)}-\ln{(1+y+z)}\right],$$ and the analogy with Beukers-type integrals is less obvious, because it doesn't seem this can be represented in the form analogous to (2).

I don't know whether there are some further deeper connections to Beukers-type integrals, or to its generalizations (for example Vasilenko integrals - see section 8 in https://eudml.org/doc/249095).

P.S. From (1) we have $$I_3=\frac{1}{2}\int\limits_0^1f(x)\;d\ln{x}=-\frac{1}{2}\int\limits_0^1 \ln{x}\,\frac{df(x)}{dx}\;dx,$$ where $$f(x)=\int\limits_0^1\frac{dy}{y}\left[\ln{(1+x)}+\ln{(1+y)}-\ln{(1+x+y)}\right],$$ and $$\frac{df(x)}{dx}=\int\limits_0^1 \frac{dy}{y}\left[\frac{1}{1+x}-\frac{1}{1+x+y}\right]=\frac{\ln{(2+x)}-\ln{(1+x)}}{1+x}.$$ Therefore $$I_3=-\frac{1}{2}\left[\int\limits_0^1 \frac{\ln{x}\ln{(2+x)}}{1+x}-\int\limits_0^1 \frac{\ln{x}\ln{(1+x)}}{1+x}\right]dx=-\frac{1}{2}\left[-\frac{13}{24}+\frac{1}{8}\right]\zeta(3)=\frac{5}{24}\zeta(3),$$ because $$\int\limits_0^1 \frac{\ln{x}\ln{(2+x)}}{1+x}dx=-\frac{13}{24}\zeta(3),$$ and $$\int\limits_0^1 \frac{\ln{x}\ln{(1+x)}}{1+x}dx=-\frac{1}{8}\zeta(3).$$ In the case of $I_4$, we analogously get $$I_4=\frac{1}{6}\int\limits_0^1\int\limits_0^1\frac{\ln{x}\ln{y}}{(1+x+y)^2}\left[\ln{(2+x+y)}-\ln{(1+x+y)}+\frac{1}{2+x+y}\right],$$ however further progress doesn't seem feasible.

$\endgroup$
5
  • 2
    $\begingroup$ Just like to share a (maybe trivial) observation on the beautiful integral for $\zeta(3)$ in your arxiv paper. When we slightly alter it into: $$f(n):=\frac{1}{7}\,\int_0^{\pi} x^n\,\frac{\pi-x}{\sin(x)}dx$$ we get closed forms expressed as finite series of $\zeta(2k+1)$ that are all weighted by $\pi^k$ and a rational. E.g. $f(2) = \frac12\pi\zeta(3)$, $f(3) = \frac32\pi^2\zeta(3)-\frac{93}{7}\zeta(5)$, $f(4) = 2\,\pi^3\zeta(3)-\frac{279}{14}\zeta(5)$, etc. $\endgroup$
    – Agno
    May 9, 2017 at 13:23
  • $\begingroup$ Interesting observation. How do you got them? By the way while checking f(2), due to an accidental error, I got (numerically) $$\int\limits_0^1\frac{x^2(\pi-x)}{\pi \sin{x}}dx=\sin\left(\frac{13\pi}{46}\right)-\sin\left(\frac{6\pi}{53}\right).$$ How this identity can be proved? $\endgroup$ May 10, 2017 at 5:25
  • $\begingroup$ It seems the last identity is only correct up to the precision $10^{-10}$ but not exactly: mathoverflow.net/questions/269405/interesting-identity $\endgroup$ May 10, 2017 at 7:20
  • $\begingroup$ I have used Maple to evaluate $f(n)$ and it came back with the closed forms. Not exactly sure how it does the evaluation, however I have the strong impression that it first expresses the integrals as a finite series of weighted polylogarithms (+other some components) that only for this specific integrals can be simplified and reduced to the form with only weighted $\zeta(2k+1)$s. $\endgroup$
    – Agno
    May 10, 2017 at 9:13
  • $\begingroup$ I have changed my comment into a follow-up question here: mathoverflow.net/questions/270286/… $\endgroup$
    – Agno
    May 21, 2017 at 12:17

Your Answer

By clicking “Post Your Answer”, you agree to our terms of service and acknowledge you have read our privacy policy.

Not the answer you're looking for? Browse other questions tagged or ask your own question.